logo

Lp空間、ルベーグ空間 📂ルベーグ空間

Lp空間、ルベーグ空間

定義1 2 3

$\Omega \subset \mathbb{R}^{n}$を開集合、$p$を正の実数としよう。

$\Omega$上で定義された全ての可測関数 $f$に対して、集合 $L^{p}(\Omega)$を以下のように定義する。

$$ L^{p}(\Omega) := \left\{ f : \int_{\Omega} \left| f(x) \right|^{p} dx < \infty \right\} $$

これをLp空間あるいはルベーグ空間と呼び、簡単に$L^{p}$などと表記することもある。通常、関数解析の教科書では上記のように記述され、測度論、実解析の教科書では次のように記述される。

測度空間 $(X, \mathcal{E}, \mu)$が与えられたとする。$X$上で定義された可測関数 $f$に対して、集合 $L^{p}(X, \mathcal{E},\mu)$を次のように定義する。

$$ L^{p}(X, \mathcal{E}, \mu) := \left\{ f : \int \left| f \right|^{p} d \mu < \infty \right\} $$

ここで$\mu$は測度である。簡単に$L^{p}(\mu), L^{p}(X)$などと表記される。

性質

  1. $L^{p}$はベクトル空間である。
  2. $1 \le p \le \infty$に対して$L^{p}$はノルム空間である。
  3. $L^{p}$は完備空間である。
  4. $E\subset X$に対して、$1 \le p \le q \le \infty$かつ$\mu (E) < \infty \implies L^{q} (E) \subset L^{p} (E)$である。

説明

2. $p \lt 1$の場合、$\left\| \cdot \right\|_{p}$は三角不等式を満たさず、ノルムにならない。しかし、$p = \infty$の場合、$L^{p}$空間はノルム空間になる

完備なノルムベクトル空間を特にバナッハ空間と呼ぶ。したがって、$L^{p}$空間はバナッハ空間である。$L^{p}$は、ヘルダーの不等式およびミンコフスキーの不等式が成立する空間として特に重要である。

内積が定義されたベクトル空間を内積空間と言う。完備な内積空間を特にヒルベルト空間と言う。$L^{2}$空間の場合は、次のように内積を定義することができる

$$ \left( \int |f(x)|^2 dx\right)^{\frac{1}{2}} = \left( \int f(x)\overline{f(x)}dx \right) ^{\frac{1}{2}} = \langle f,f \rangle ^{\frac{1}{2}} $$

したがって、$L^{2}$空間はヒルベルト空間である。

4. $\mu (E) < \infty$という条件に注目しよう。もし積分範囲が有界でない場合、$L^{1} (E)$と$L^{2} (E)$はいかなる包含関係も持たなくなる。$1 \le p \lt q \lt r$が特定の条件を満たす場合、${u \in L^{p} \cap L^{r} \implies u \in L^{q}}$が成り立つこともある

証明

2.

$1\le p <\infty$に対して$\| \cdot \|_{p}$を次のように定義する。

$$ \left\| f \right\|_{p} := \left( \int_{\Omega} \left| f(x) \right|^{p} dx \right)^{1/p},\quad f\in L^{p}(\Omega) $$

すると、$\| \cdot \|_{p}$は$L^{p}$空間のノルムになる($0<p<1$の時はノルムにならない)。$\| f \|_{p} \ge 0$であることは自明であり、$\| f \|_{p}=0 \iff f=0$であることも自明だ。また、$c \in \mathbb{C}$に対して$\| cf \|_{p} = \left| c \right| \left\| f \right\|_{p}$が成立することも以下のように示すことができる。

$$ \begin{align*} \left\| cf \right\|_{p} =& \left( \int_{\Omega} \left| cf(x) \right|^{p} dx \right)^{1/p} \\ =& \left( \left| c \right|^{p} \int_{\Omega} \left| f(x) \right|^{p} dx \right)^{1/p} \\ =& \left| c \right| \left( \int_{\Omega} \left| f(x) \right|^{p} dx \right)^{1/p} \\ =& \left| c \right| \left\| f \right\|_{p} \end{align*} $$

$f,g \in L^{p}$に対して、$\left\| f + g \right\|_{p} \le \| f \|_{p} + \| g \|_{p}$も同様に成立し、これはミンコフスキーの不等式と呼ばれている。

3.

戦略:ほとんど全てがファトゥの補題によって解決される。


与えられたコーシー数列 $f_{n}$ に対して、$\left\| f_{n} - f_{n_{k}} \right\|_{p} < \dfrac{1}{2^{k}}$を満たす部分数列 $f_{n_{k}}$ を見つけることができる。全ての $k \in \mathbb{N}$ に対して

$$ \begin{align*} g_{k} :=& \sum_{i=1}^{k} \left| f_{n_{i+1}} - f_{n_{i}} \right| \\ g :=& \lim_{k \to \infty} g_{k} = \sum_{i=1}^{\infty} \left| f_{n_{i+1}} - f_{n_{i}} \right| \end{align*} $$

を定義すると、三角不等式によって

$$ \left\| g_{k} \right\|_{p} \le \sum_{i}^{k} \dfrac{1}{2^{i}} < 1 $$

ファトゥの補題

関数値が非負の可測関数数列 $\left\{ f_{n} \right\}$ に対して

$$ \int \left( \liminf_{n \to \infty} f_{n} \right) d \mu \le \liminf_{n \to \infty} \int f_{n} d \mu $$

ファトゥの補題により

$$ \left\| g \right\|_{p}^{p} \le \int \lim_{n \to \infty} g_{k}^{p} d \mu \le \liminf_{k \to \infty} \int g_{k}^{p} d \mu \le 1 $$

$g$がほとんど至る所で有限であるため、

$$ f_{n_{k}} = f_{n_{1}}(x) + \sum_{i=1}^{ k } \left[ f_{n_{i}} (x) - f_{n_{i-1}} (x) \right] $$

ほとんど至る所で収束する。$f := \lim\limits_{k \to \infty} f_{n_{k}}$と定義すると、ファトゥの補題により

$$ \left\| f - f_{m} \right\|_{p} = \int |f - f_{m}|^{p} d \mu \le \liminf_{k \to \infty} \int | f_{n_{k}} - f_{m}|^{p} d \mu \le \varepsilon^{p} $$

したがって$f - f_{m} \in L^{p}$であり、$f = f_{m} + (f - f_{m} ) \in L^{p}$である。$L^{p}$の全てのコーシー数列が$L^{p}$の元に収束するため、$L^{p}$は完備空間である。

4.

戦略:$|f(x)|^{p} \le 1 + |f(x)|^{q}$という不等式を示せば、残りはルベーグ積分の性質によって証明が終わる。


$f \in L^{q}$としよう。すると、次の式が成り立つ。

$$ \begin{align*} | f(x) | \le 1 \implies& |f(x) |^{p} \le 1 \\ 1 \le |f(x)| \implies& |f(x)|^{p} \le |f(x)|^{q} \end{align*} $$

したがって、$| f(x) |$が$1$より大きいか小さいかにかかわらず、次が成り立つ。

$$ |f(x)|^{p} \le 1 + |f(x)|^{q} $$

ルベーグ積分 $\displaystyle \int_{E} d \mu$ をとると、以下のようになる。

$$ \int_{E} |f|^{p} d \mu \le \int_{E} 1 d \mu + \int_{E} |f|^{q} d \mu = m(E) + \int_{E} |f|^{q} d \mu < \infty $$

$m(E) < \infty$であり$\displaystyle \int_{E} |f|^{q} d \mu < \infty$であるため、次が成り立つ。

$$ \int_{E} |f|^{p} d \mu < \infty $$

言い換えれば、$f \in L^{q} \implies f \in L^{p}$であるため、

$$ L^{q} (E) \subset L^{p} (E) $$

参照


  1. Capinski, Measure, Integral and Probability (1999), p140 ↩︎

  2. Robert A. Adams and John J. F. Foutnier, Sobolev Space (2nd Edition, 2003), p23 ↩︎

  3. Gerald B. Folland, Real Analysis: Modern Techniques and Their Applications (2nd Edition, 1999), p181 ↩︎